LSAT and Law School Admissions Forum

Get expert LSAT preparation and law school admissions advice from PowerScore Test Preparation.

 Administrator
PowerScore Staff
  • PowerScore Staff
  • Posts: 8927
  • Joined: Feb 02, 2011
|
#24970
Complete Question Explanation

Weaken—CE. The correct answer choice is (D)

In this Weaken stimulus, the medical researcher presents the results of a study on jogging. The conclusion is causal and can be diagramed as follows:

  • Cause ..... ..... ..... ..... ..... ..... ..... Effect

    stretching prior to jogging ..... :arrow: ..... prevent jogging injuries

It is based on the results of a study involving a large number of joggers, split into equal groups of stretchers and non-stretchers. During the study the same number of joggers from each group experienced jogging related injuries. Thus, the researcher concluded that stretching prior to jogging did not actually help to prevent injuries. To weaken this conclusion, we will want to attack the study. The groups are the same size, and the structure of the study does not provide a motivation for participants to report false data. While this question may be a bit difficult to prephrase, we are still able to recognize that the correct answer choice will likely be related to the casual relationship described above.

Answer choice (A): This answer choice is incorrect because we do not have information about the overall rate of stretching in the general jogging public. Since the conclusion focuses on a causal relationship between stretching and avoiding jogging injuries, without information regarding the frequency of stretching in the general jogging public we cannot determine how this answer choice would impact the argument.

Answer choice (B): This answer choice looks tempting, because it seems to suggest that the injuries cause the difficulty with stretching. However, the conclusion does not distinguish between those who had difficulty with stretching and those who did not. It doesn’t matter if those with prior injuries struggled to stretch as long as they actually did stretch.

Answer choice (C): This answer choice actually strengthens the argument. It supports the idea that stretching does not help prevent injuries by stating that most jogging injuries are caused by factors that stretching would not address.

Answer choice (D): This is the correct answer choice. Answer choice (D) suggests that joggers who are particularly prone to injuries are more likely to stretch before jogging. Given that both groups of joggers incurred the same number of injuries, it is reasonable to suspect that stretching can lower the otherwise high risk of jogging injuries among the people who stretch, so that both groups end up with the same number of injuries. This would corroborate the theory that stretching lowers the risk of injury, and weaken the conclusion of the argument.

Answer choice (E): This answer choice is incorrect as the stimulus does not differentiate between the severity of the injuries sustained by the joggers. The study only counted injuries versus non-injuries. Therefore, the fact that stretching can reduce the severity of injuries does not impact the causal argument relating the behavior of stretching to the occurrence of any injury.
 AylixW
  • Posts: 14
  • Joined: Jul 11, 2012
|
#5263
Hi,

I have read this question about five times and cannot figure out why D weakens the argument. I chose B because I figured if people who were injured weren't able to stretch, that weakens that stretching doesn't help prevent injuries because then the people who are injured could make up a large amount of people who weren't stretching and were the ones continuously being injured. Therefore, it is not proven that stretching does not do anything since a majority isn't stretching. Hopefully that kind of makes sense, but if you could please explain why the answer is D, that would be great!

Thanks,
Aylix
 Steve Stein
PowerScore Staff
  • PowerScore Staff
  • Posts: 1153
  • Joined: Apr 11, 2011
|
#5273
Hi Aylix,

That's a good question; The author discusses the comparison between two equal sized groups of joggers--stretchers and non-stretchers--and their equal number of injuries. The author concludes that stretching does no good preventing injuries.

Again, the conclusion is that stretching doesn't prevent injuries, and the stimulus is followed by a Weaken question. Correct answer D provides that joggers who are more likely to get injured are also more likely to stretch. That group already had just as many injuries as the non-stretchers, but if they didn't stretch, according to this answer choice, it would have been even worse.

In other words, according to answer choice D, the group of stretchers started out at a disadvantage--they were more likely to injure themselves from the outset. Stretching kept their injuries as low as the other group, so this choice weakens the author's conclusion, by providing evidence that stretching actually does do some good with regard to injury prevention.

I hope that's helpful--let me know--thanks!

~Steve
 avengingangel
  • Posts: 275
  • Joined: Jun 14, 2016
|
#30479
Although I did not choose E, I am wondering: What if the study mentioned in E addressed the frequency of injuries (versus severity), therefore addressing the same thing that the stimulus' study is, and also said "does not reduce?" In general, what is the role of "other studies" mentioned in answer choices? Can that ever be a correct answer to a strengthen/weaken question??
 Adam Tyson
PowerScore Staff
  • PowerScore Staff
  • Posts: 5153
  • Joined: Apr 14, 2011
|
#30557
The problem with answer E, angel, is that even if we altered the answer to talk about number rather than severity of injuries, the conclusion is specifically about jogging - stretching before jogging does not help to prevent injuries. What effect stretching may or may not have in other sports simply doesn't matter, because the conclusion is limited to jogging. Of course, if it said that stretching does not reduce the number of injuries, that might strengthen the argument, showing that the same results holds.

Reference to other studies can often help or hurt an argument, if the references are relevant. For example, if you have a stimulus that says 4 studies were done that supported the hypothesis that wolves are primarily carnivorous, and you wanted to weaken that claim about wolves, a 5th study that showed that wolves were primarily vegetarian would weaken the claim. A study of coyotes, or of mountain lions, would not have any impact. Nor would a study of wolves sleeping habits, or a study that showed that in the absence of a meat supply wolves will eat fruits and veggies. It all depends on what those other studies show.

There are no blanket rules to tell you when it's okay to accept other studies and when to reject them. It will always depend on context.

You've obviously been very busy at your studies - keep it up!
 avengingangel
  • Posts: 275
  • Joined: Jun 14, 2016
|
#30574
That was really helpful. I don't think I realized that answer choice E said "for certain forms of exercise," so thanks for pointing that out. And the examples of different kinds of studies make a lot of sense and show how sometimes they can hurt/help and sometimes they are irrelevant.

Thanks Adam!
 RJF
  • Posts: 6
  • Joined: Jun 29, 2017
|
#39143
Hello,

When reading answer choice D), it felt like something was missing:

Firstly, it seems that we have to assume that, because the joggers (that are prone to injuries) have developed the habit of stretching, that this habit has indeed paid off (i.e., prevented potential injury). Because of this assumption (that the newly-formed habit of stretching has been successful), I dismissed answer choice D), thinking that the newly-formed habit of stretching didn't necessarily have to have been successful - that I couldn't just assume that it had been successful.

Secondly, what if the joggers that are used to stretching were in already in the 'Stretchers' group? If they were in the 'Non-stretchers' group, D) would make sense: i.e., that joggers who must stretch (for pre-determined reasons) were unable to do so because they were placed in the 'Non-stretchers' group, so naturally numbers would be high.

I must be over thinking this. Can anyone please help?

Thank you.
 AthenaDalton
PowerScore Staff
  • PowerScore Staff
  • Posts: 296
  • Joined: May 02, 2017
|
#39430
Hi RJF,

Thanks for your question!

I think you are over-thinking the group design a bit. We aren't told that people who would normally stretch were instructed not to stretch, so don't make that assumption. :)

You will see this sort of setup frequently on the LSAT. To draw conclusions about the utility (or lack thereof) of stretching between two different groups, we first need to know that the groups are the same. Answer choice (D) should jump out because it tells us that there was a really important difference between the stretching and non-stretching groups -- people who stretch do so because they know they are at risk of getting injured!

So just to give an extreme example, imagine that Group 1 is made up of novice runners with severe osteoporosis, balance issues, and little experience running. Group 2 is made up of runners training for the Olympics. If Group 1 stretches and Group 2 does not, can we really say that stretching provided no value? It's hard to make any conclusion about the utility of stretching between two such different groups. If anything, it looks like stretching did benefit Group 1 because we would expect them to have a lot of injuries compared to Group 2, but in fact they had fewer injuries than expected.

A properly-designed experiment would involve splitting two identical groups of runners (in terms of age, experience, injury history) into two groups and assigning one group pre-run stretches and the other group no stretches. Any study design that deviates from this gold standard should make you suspicious. :)

I hope this helps clarify things for you. Good luck studying!

Athena
 Toby
  • Posts: 33
  • Joined: Jun 05, 2017
|
#47354
Hello!

I have a question about what's exactly going on in this stimulus. Are the medical researchers dividing the group of joggers into two smaller groups, asking one group of joggers to stretch, asking the other group of joggers not to stretch, and then observing what happens to them after they go for a run? Or are the medical researchers just gathering information from these two groups without giving them a "treatment" (ie: jogging)? Thanks!

Toby
 Adam Tyson
PowerScore Staff
  • PowerScore Staff
  • Posts: 5153
  • Joined: Apr 14, 2011
|
#47374
The researchers aren't dividing the joggers into groups, Toby, but rather the joggers are dividing themselves into groups. That's part of what makes the study suspect - the self-selecting nature of the sample groups could mean that there is an important difference between them, and that difference comes out in the correct answer choice. A better study would have had the researchers randomly divide the joggers into two groups, telling one to do stretches and the other not to do stretches. Important difference!

Get the most out of your LSAT Prep Plus subscription.

Analyze and track your performance with our Testing and Analytics Package.